Câu hỏi:

Giới hạn \(\mathop {\lim }\limits_{n \to  + \infty } \dfrac{{1 + 2 + 3 + ... + \left( {n - 1} \right) + n}}{{{n^2}}}\) bằng

  • A \( + \infty \)   
  • B \(1\)
  • C \(0\)
  • D \(\frac{1}{2}\)

Phương pháp giải:

Sử dụng quy tắc tính giới hạn của dãy số để tính.

Lời giải chi tiết:

Ta có: \(\mathop {\lim }\limits_{x \to \infty } \frac{{1 + 2 + 3 + .... + \left( {n - 1} \right) + n}}{{{n^2}}} = \mathop {\lim }\limits_{x \to \infty } \frac{{\frac{{n\left( {n + 1} \right)}}{2}}}{{{n^2}}} = \mathop {\lim }\limits_{x \to \infty } \frac{{{n^2} + n}}{{2{n^2}}} = \frac{1}{2}.\)

Chọn  D.


Quảng cáo

Luyện Bài Tập Trắc nghiệm Toán 11 - Xem ngay